Difference between revisions of "2018 AMC 10A Problems/Problem 1"

(Video Solution 2)
(Video Solution 2)
Line 17: Line 17:
 
==Video Solution==
 
==Video Solution==
 
https://youtu.be/vO-ELYmgRI8
 
https://youtu.be/vO-ELYmgRI8
 
==Video Solution 2==
 
https://youtu.be/Ns34Jiq9ofc
 
—DSA_Catachu
 
  
 
== See Also ==
 
== See Also ==

Revision as of 15:35, 1 May 2020

Problem

What is the value of \[\left(\left((2+1)^{-1}+1\right)^{-1}+1\right)^{-1}+1?\] $\textbf{(A) } \frac58 \qquad \textbf{(B) }\frac{11}7 \qquad \textbf{(C) } \frac85 \qquad \textbf{(D) } \frac{18}{11} \qquad \textbf{(E) } \frac{15}8$

Solution

\[\left(\left((2+1)^{-1}+1\right)^{-1}+1\right)^{-1}+1\] \[=\left(\left(3)^{-1}+1\right)^{-1}+1\right)^{-1}+1\] \[=\left(\left(\frac{1}{3}+1\right)^{-1}+1\right)^{-1}+1\] \[=\left(\left(\frac{4}{3}\right)^{-1}+1\right)^{-1}+1\] \[=\left(\frac{3}{4}+1\right)^{-1}+1\] \[=\left(\frac{7}{4}\right)^{-1}+1\] \[=\frac{4}{7}+1\] \[=\frac{11}{7}\] Therefore, the answer is $\boxed{\textbf{(B) } \frac{11}{7} }$.

Video Solution

https://youtu.be/vO-ELYmgRI8

See Also

2018 AMC 10A (ProblemsAnswer KeyResources)
Preceded by
First Problem
Followed by
Problem 2
1 2 3 4 5 6 7 8 9 10 11 12 13 14 15 16 17 18 19 20 21 22 23 24 25
All AMC 10 Problems and Solutions

The problems on this page are copyrighted by the Mathematical Association of America's American Mathematics Competitions. AMC logo.png